Difference between revisions of "1964 AHSME Problems/Problem 16"

(Solution)
(Solution $2$=)
Line 10: Line 10:
 
\textbf{(E)}\ 17  </math>
 
\textbf{(E)}\ 17  </math>
  
==Solution==
+
==Solution <math>1</math>==
  
 
Note that for all polynomials <math>f(x)</math>, <math>f(x + 6) \equiv f(x) \pmod 6</math>.   
 
Note that for all polynomials <math>f(x)</math>, <math>f(x + 6) \equiv f(x) \pmod 6</math>.   
Line 20: Line 20:
  
 
This means for <math>s=1, 2, 4, 5</math>, <math>f(s)</math> is divisible by <math>6</math>.  Since <math>f(1)</math> is divisible, so is <math>f(s)</math> for <math>s=7, 13, 19, 25</math>, which is <math>5</math> values of <math>s</math> that work.  Since <math>f(2)</math> is divisible, so is <math>f(s)</math> for <math>s=8, 14, 20</math>, which is <math>4</math> more values of <math>s</math> that work.  The values of <math>s=4, 5</math> will also generate <math>4</math> solutions each, just like <math>f(2)</math>.  This is a total of <math>17</math> values of <math>s</math>, for an answer of <math>\boxed{\textbf{(E)}}</math>
 
This means for <math>s=1, 2, 4, 5</math>, <math>f(s)</math> is divisible by <math>6</math>.  Since <math>f(1)</math> is divisible, so is <math>f(s)</math> for <math>s=7, 13, 19, 25</math>, which is <math>5</math> values of <math>s</math> that work.  Since <math>f(2)</math> is divisible, so is <math>f(s)</math> for <math>s=8, 14, 20</math>, which is <math>4</math> more values of <math>s</math> that work.  The values of <math>s=4, 5</math> will also generate <math>4</math> solutions each, just like <math>f(2)</math>.  This is a total of <math>17</math> values of <math>s</math>, for an answer of <math>\boxed{\textbf{(E)}}</math>
 +
 +
==Solution <math>2</math>==
 +
 +
We have <math>f(</math>x<math>)</math>=<math>x</math>^<math>2</math> + <math>3</math><math>x</math> +<math>2</math>  and <math>S</math> = {<math>0</math>,<math>1</math>,<math>2</math>,... ,<math>25</math>}.We need equiv <math>f</math>(<math>s</math>) \pmod 6<math>. T hat implies we need numbers of the form </math>6<math></math>k<math> +</math>5<math> and </math>6<math></math>k<math> +</math>4<math> also due to restriction on number of elements of set we need </math>k<math><= 3 . Calculate the number of possibble combinations to get </math>17<math> as answer.</math>Ans.<math> </math>\boxed{\textbf{(E)}}<math>.
 +
 +
Solution by 4GEOMETRY-WIZARD</math>.
  
 
==See Also==
 
==See Also==

Revision as of 03:21, 31 December 2023

Problem

Let $f(x)=x^2+3x+2$ and let $S$ be the set of integers $\{0, 1, 2, \dots , 25 \}$. The number of members $s$ of $S$ such that $f(s)$ has remainder zero when divided by $6$ is:

$\textbf{(A)}\ 25\qquad \textbf{(B)}\ 22\qquad \textbf{(C)}\ 21\qquad \textbf{(D)}\ 18 \qquad \textbf{(E)}\ 17$

Solution $1$

Note that for all polynomials $f(x)$, $f(x + 6) \equiv f(x) \pmod 6$.

Proof: If $f(x) = a_nx^n + a_{n-1}x^{n-1} + ... + a_0$, then $f(x+6) = a_n(x+6)^n + a_{n-1}(x+6)^{n-1} +...+ a_0$. In the second equation, we can use the binomial expansion to expand every term, and then subtract off all terms that have a factor of $6^1$ or higher, since subtracting a multiple of $6$ will not change congruence $\pmod 6$. This leaves $a_nx^n + a_{n-1}x^{n-1} + ... + a_0$, which is $f(x)$, so $f(x+6) \equiv f(x) \pmod 6$.

So, we only need to test when $f(x)$ has a remainder of $0$ for $0, 1, 2, 3, 4, 5$. The set of numbers $6, 7, 8, 9, 10, 11$ will repeat remainders, as will all other sets. The remainders are $2, 0, 0, 2, 0, 0$.

This means for $s=1, 2, 4, 5$, $f(s)$ is divisible by $6$. Since $f(1)$ is divisible, so is $f(s)$ for $s=7, 13, 19, 25$, which is $5$ values of $s$ that work. Since $f(2)$ is divisible, so is $f(s)$ for $s=8, 14, 20$, which is $4$ more values of $s$ that work. The values of $s=4, 5$ will also generate $4$ solutions each, just like $f(2)$. This is a total of $17$ values of $s$, for an answer of $\boxed{\textbf{(E)}}$

Solution $2$

We have $f($x$)$=$x$^$2$ + $3$$x$ +$2$ and $S$ = {$0$,$1$,$2$,... ,$25$}.We need equiv $f$($s$) \pmod 6$. T hat implies we need numbers of the form$6$$ (Error compiling LaTeX. Unknown error_msg)k$+$5$and$6$$ (Error compiling LaTeX. Unknown error_msg)k$+$4$also due to restriction on number of elements of set we need$k$<= 3 . Calculate the number of possibble combinations to get$17$as answer.$Ans.$$ (Error compiling LaTeX. Unknown error_msg)\boxed{\textbf{(E)}}$.

Solution by 4GEOMETRY-WIZARD$ (Error compiling LaTeX. Unknown error_msg).

See Also

1964 AHSC (ProblemsAnswer KeyResources)
Preceded by
Problem 15
Followed by
Problem 17
1 2 3 4 5 6 7 8 9 10 11 12 13 14 15 16 17 18 19 20 21 22 23 24 25 26 27 28 29 30 31 32 33 34 35 36 37 38 39 40
All AHSME Problems and Solutions

The problems on this page are copyrighted by the Mathematical Association of America's American Mathematics Competitions. AMC logo.png